Integrating a Square Root Function: Solution

Click For Summary
The discussion focuses on solving the integral ∫(0,1) √x/√[3]1-x using the beta and gamma functions. The user attempts to apply the recursive properties of the gamma function but struggles with the calculations, particularly in determining the values of β(3/2, 2/3) and the associated gamma functions. There is confusion regarding the correct application of the recursive formula and the relationships between the gamma functions. The conversation highlights the challenge of finding analytical expressions for certain integrals, suggesting that numerical approximations may be necessary. Overall, the thread emphasizes the complexities involved in integrating functions involving square roots and gamma functions.
Ikastun
Messages
10
Reaction score
0

Homework Statement



∫(0,1) √x/√[3]1-x

Homework Equations



\Gammap\Gammaq/\Gammap+q

The Attempt at a Solution



p-1=1/2 →p=3/2
q-1=-1/3 →q=2/3

β(3/2,2/3)=\Gamma(3/2) \Gamma(2/3)/\Gamma(13/6)

\Gamma3/2=1/2\Gamma(1/2)=√π/2
\Gamma2/3=-1/3
\Gamma13/6=7/6 1/6=7/36

β(3/2,2/3)=-6√π/7
 
Physics news on Phys.org
Ikastun said:
\Gamma2/3=-1/3
\Gamma13/6=7/6 1/6=7/36
That looks wrong.

Can you explain what you want to calculate, how you attempt to do this and where your problem is?
 
Hello and thank you for answering.

My problem begins with the part you quote. I don't know how to properly use the recursive formula in those cases.
Regarding my attempt to calculate the integral, what I wrote above is everything.
 
##\Gamma(\frac{13}{6})=\frac{7}{6}\Gamma(\frac{7}{6}) =\frac{7}{36}\Gamma(\frac{1}{6})##
For some values, an analytic expression is known, in general this doesn't work and you have to live with the expressions (or find a numerical approximation).

Regarding my attempt to calculate the integral, what I wrote above is everything.
There is some connections between the formulas that you could explain.
 
Question: A clock's minute hand has length 4 and its hour hand has length 3. What is the distance between the tips at the moment when it is increasing most rapidly?(Putnam Exam Question) Answer: Making assumption that both the hands moves at constant angular velocities, the answer is ## \sqrt{7} .## But don't you think this assumption is somewhat doubtful and wrong?

Similar threads

  • · Replies 2 ·
Replies
2
Views
2K
Replies
12
Views
2K
  • · Replies 7 ·
Replies
7
Views
1K
  • · Replies 18 ·
Replies
18
Views
5K
  • · Replies 3 ·
Replies
3
Views
2K
  • · Replies 10 ·
Replies
10
Views
2K
  • · Replies 9 ·
Replies
9
Views
2K
Replies
3
Views
2K
  • · Replies 10 ·
Replies
10
Views
2K
Replies
7
Views
1K